Limit proof as x approaches infinity

Click For Summary
SUMMARY

The discussion centers on verifying the assertion that \(x^2 + \sqrt{x} = O(x^2)\) as \(x\) approaches infinity. The participants utilize the limit definition of Big O notation, specifically that if the limit of \(\frac{f(x)}{g(x)}\) exists and is finite, then \(f(x) = O(g(x))\). Through manipulation, they conclude that \(\frac{x^2 + \sqrt{x}}{x^2} = 1 + \frac{1}{x^{3/2}}\) leads to the result that \(x^2 + \sqrt{x} \leq k x^2\) for \(k = 2\). This confirms the assertion with a finite constant, adhering to the formal definition of Big O notation.

PREREQUISITES
  • Understanding of Big O notation in algorithm analysis
  • Familiarity with limits and asymptotic analysis
  • Basic algebraic manipulation skills
  • Knowledge of calculus concepts, particularly limits as \(x\) approaches infinity
NEXT STEPS
  • Study the formal definition of Big O notation and its applications in algorithm complexity
  • Learn how to apply limits in asymptotic analysis using L'Hôpital's Rule
  • Explore other examples of proving functions are \(O(g(x))\) using different methods
  • Investigate the differences between Big O, Big Theta, and Big Omega notations
USEFUL FOR

Students in computer science, mathematicians focusing on algorithm analysis, and anyone interested in understanding asymptotic behavior of functions in computational contexts.

fishturtle1
Messages
393
Reaction score
82

Homework Statement


Verify the following assertions:
a) ##x^2 + \sqrt{x} = O(x^2)##

2. Homework Equations

If the limit as x approaches ##\infty## of ##\frac {f(x)}{g(x)}## exists (and is finite), then ##f(x) = O(g(x))##.

The Attempt at a Solution


Let ##\epsilon > 0##. We solve for ##\delta## such that ##x > \delta## implies ##|\frac{f(x)}{g(x)} - 1| < \epsilon##. Consider ##|\frac {x^2 + \sqrt{x}}{x^2} - 1| < \epsilon##. Then ##\frac {x^2 + \sqrt{x}}{x^2} - 1 < \epsilon##, so ##\frac {x^2 + \sqrt{x}}{x^2} < \epsilon + 1##. Taking the inverse of both sides and then taking the square root of both sides, we get ##\frac x{\sqrt{\sqrt{x} + x^2}} > \frac 1{\sqrt{\epsilon + 1}}##..I'm trying to get to an expression "x > some expression in terms of ##\epsilon##" and then substitute that for ##\delta##.
 
Physics news on Phys.org
Why would you take the inverse square root of both sides?
$$\frac {x^2 + \sqrt{x}}{x^2} < \epsilon + 1$$That is good. Now simplify it.
 
mfb said:
Why would you take the inverse square root of both sides?
$$\frac {x^2 + \sqrt{x}}{x^2} < \epsilon + 1$$That is good. Now simplify it.
Thanks for the response,

##\frac {x^2 + \sqrt{x}}{x^2} = 1 + \frac{\sqrt{x}}{{x^2}} = 1 + \frac 1{x^{3/2}} < \epsilon + 1## so, ##\frac 1{x^{3/2}} < \epsilon##
Raising both sides to ##-5/2##, we get ##x > \epsilon^{\frac{-5}{2}} = \delta##. Thus, we can conclude the limit as x approaches ##\infty## for ##\frac {x^2 + \sqrt{x}}{x^2} = 1##. Thus, ## {x^2 + \sqrt{x}} = O(x^2)##.
 
Last edited:
fishturtle1 said:

Homework Statement


Verify the following assertions:
a) ##x^2 + \sqrt{x} = O(x^2)##

2. Homework Equations

If the limit as x approaches ##\infty## of ##\frac {f(x)}{g(x)}## exists (and is finite), then ##f(x) = O(g(x))##.

The Attempt at a Solution


Let ##\epsilon > 0##. We solve for ##\delta## such that ##x > \delta## implies ##|\frac{f(x)}{g(x)} - 1| < \epsilon##. Consider ##|\frac {x^2 + \sqrt{x}}{x^2} - 1| < \epsilon##. Then ##\frac {x^2 + \sqrt{x}}{x^2} - 1 < \epsilon##, so ##\frac {x^2 + \sqrt{x}}{x^2} < \epsilon + 1##. Taking the inverse of both sides and then taking the square root of both sides, we get ##\frac x{\sqrt{\sqrt{x} + x^2}} > \frac 1{\sqrt{\epsilon + 1}}##..I'm trying to get to an expression "x > some expression in terms of ##\epsilon##" and then substitute that for ##\delta##.
You have
$$\frac{x^2+\sqrt{x}}{x^2} = 1 + \frac{1}{x^{3/2}} \leq 2$$
for ##x \geq 1##. Thus, ##x^2 + \sqrt{x} \leq k x^2## where ##k = 2##. If you further restrict the range of ##x > 0## you can make ##k## smaller, but that is not relevant to the definition of ##O(x^2)##, which just need some finite ##k > 0##.

BTW: your definition of ##O## is different from those I have seen; the ones I know of do not require that ##f(x)/g(x)## have a finite limit as ##x \to \infty##, they just require that there exists some finite ##k > 0## such that ##|f(x)/g(x)| \leq k## as ##x \to \infty##.
 
Ray Vickson said:
You have
$$\frac{x^2+\sqrt{x}}{x^2} = 1 + \frac{1}{x^{3/2}} \leq 2$$
for ##x \geq 1##. Thus, ##x^2 + \sqrt{x} \leq k x^2## where ##k = 2##. If you further restrict the range of ##x > 0## you can make ##k## smaller, but that is not relevant to the definition of ##O(x^2)##, which just need some finite ##k > 0##.

BTW: your definition of ##O## is different from those I have seen; the ones I know of do not require that ##f(x)/g(x)## have a finite limit as ##x \to \infty##, they just require that there exists some finite ##k > 0## such that ##|f(x)/g(x)| \leq k## as ##x \to \infty##.
Thanks for the clarification, I didn't know about the actual definition of ##O##.

I should add that the textbook says this method can *sometimes* be used to prove that ##f(x) = O(g(x))##. So I guess the book's method only works for certain situations.. and even if it fails, it doesn't rule out whether or not ##f(x) = O(g(x))##.
 
You showed more than just O(x2), but showing that the limit is 1 shows the existence of the limit, of course.
 
  • Like
Likes fishturtle1
Question: A clock's minute hand has length 4 and its hour hand has length 3. What is the distance between the tips at the moment when it is increasing most rapidly?(Putnam Exam Question) Answer: Making assumption that both the hands moves at constant angular velocities, the answer is ## \sqrt{7} .## But don't you think this assumption is somewhat doubtful and wrong?

Similar threads

  • · Replies 31 ·
2
Replies
31
Views
2K
Replies
2
Views
2K
Replies
6
Views
2K
  • · Replies 14 ·
Replies
14
Views
2K
  • · Replies 2 ·
Replies
2
Views
1K
  • · Replies 1 ·
Replies
1
Views
1K
Replies
16
Views
2K
  • · Replies 13 ·
Replies
13
Views
4K
  • · Replies 2 ·
Replies
2
Views
2K
  • · Replies 9 ·
Replies
9
Views
4K